Editorial: This political party has repeatedly expressed the view that increasing spending on education is a worth...

Bly2828 on December 17, 2018

Help

I was torn between A and D for this question. Can someone please elaborate as to why A is incorrect?

Reply
Create a free account to read and take part in forum discussions.

Already have an account? log in

Ravi on December 19, 2018

@Bly2828,

This is a tricky question, and (A) and (D) are definitely the most popular answer choices. This is a Strengthen with a Necessary Premise question.

(A) sounds good at first, but if you read it closely, you'll see that it says "legislators" and "vote against." Our stimulus provides us with no information about legislators or voting of any kind; rather, it discusses political parties and their expressed viewpoints. (A) is baiting us to draw connections between political parties/legislators as well as voting against/the expression of views. However, we can't make these connections without support, and the stimulus gives us no support to make such connections. Because of this, we can eliminate (A).

The best test for eliminating (A) is the negation test that we use for all strengthen with necessary premise questions. If we negate (A), it does not wreck the argument because as mentioned above, the stimulus says nothing about legislators or voting against.

(D) is definitely essential for this argument. (D) discusses political parties holding an opinion about something, and this is exactly what we have in the stimulus. If we were to negate (D), then a consistent political party could hold that an action that comprises a worthy goal should not be performed. If this negation were true, it would make the argument fall apart because the conclusion is saying the opposite.